হকিং রেডিয়েশন কীভাবে ব্ল্যাক হোলের ভর হ্রাস করে?


33

আমি এতক্ষণ যা বুঝতে পেরেছি সেগুলি থেকে যখন ভার্চুয়াল কণাগুলির একটি ইভেন্টের দিগন্তকে অতিক্রম করে এবং অন্যটি না করে, তারা একে অপরকে ধ্বংস করতে পারে না। পরেরটি মহাবিশ্বের দিকে বিভ্রান্ত হয় (বিটিডব্লিউ, এটি কি এখনও এই স্থানে ভার্চুয়াল, এবং 'ভার্চুয়াল' এর অর্থ এই মুহুর্তে কী, যদি তাই হয়?), অন্যটি ব্ল্যাকহোল দ্বারা গ্রাস হয়ে যায়। আমি দেখতে পাচ্ছি না যে এই ঘটনাটি ব্ল্যাকহোলের বাষ্পীভবনে ভূমিকা রাখে (যেহেতু ব্ল্যাকহোল থেকে কণাগুলি উত্পন্ন হয় না)। গ্রাসকৃত কণাটি কি ব্ল্যাকহোলের ভরগুলিতে আসলে যুক্ত করা উচিত নয়?

আমার নিকটতম প্রশ্ন হ'ল হকিংয়ের বিকিরণগুলি কি মহাবিশ্বে ভর করে? , তবে উত্তরগুলি সন্তোষজনক মনে করি না।

অর্থাৎ " পালিয়ে যাওয়া ভার্চুয়াল কণাকে ব্ল্যাকহোলের মহাকর্ষীয় ক্ষেত্রটি একটি সত্যিকারের কণায় পরিণত করার" দ্বারা উত্সাহিত করা হয়েছে ", বরং প্রশ্নটি যুক্ত করে তার উত্তর দিন।

সম্পাদনা: আমি উত্তরে উপস্থাপিত জ্ঞান দ্বারা নম্র হয়েছি এবং যেকোনকে সবচেয়ে উপযুক্ত হিসাবে চিহ্নিত করতে অপারগ বোধ করছি। আমি আশা করি এটি ঠিক আছে।


9
ভার্চুয়াল কণা জোড়া সম্পর্কে ভুলে যান, এটি আরও একটি রূপকের মতো। জিনিসটি হ'ল, ব্ল্যাক হোলগুলি কীভাবে তা করে তা নির্বিশেষে বিকিরণ নির্গত করে। বিকিরণ শক্তি বহন করে, যা কোথাও থেকে আসতে হবে, এখানে কোনও নিখরচায় দুপুরের খাবার নেই। কিন্তু শক্তি ভর ভর। ব্ল্যাকহোলের "ব্যাঙ্ক অ্যাকাউন্ট" থেকে এটি সমস্তই বেরিয়ে আসে, কারণ এটিই নিকটেই only
ফ্লোরিন আন্দ্রেই

3
@ মার্কো ৩৩ এটি যে ভার্চুয়াল কণাগুলি সাধারণভাবে একটি রূপক নয় (যদিও এটি এক অর্থে সমস্ত কণা, ভার্চুয়াল বা অন্যথায় অন্তর্নিহিত ক্ষেত্রগুলির কিছু দিক দেখার একটি উপায়) তবে এগুলির সত্যিকারের খুব ভাল ব্যাখ্যা নয় are হকিং রেডিয়েশন এটি অবশ্য আপনার প্রশ্নের উত্তর দেয় না। আমি একটি উত্তর দেখার অপেক্ষায় রয়েছি।
স্টিভ লিন্টন

1
আপনি চেক আউট করতে পছন্দ করতে পারেন math.ucr.edu/home/baez/physics/Quantum/virtual_particles.html এবং physics.stackexchange.com/questions/185110/... ভার্চুয়াল কণা সম্পর্কে পদার্থবিজ্ঞান বিষয়ে এবং অন্যান্য সম্পর্কিত প্রশ্ন।
প্রধানমন্ত্রী 2 রিং

1
পদার্থবিজ্ঞানের উপর এই প্রশ্নটি (এবং এটির স্বীকৃত উত্তরটি এখনও এই প্রশ্নের সমাধান করার জন্য আমি কাছে পেয়েছি) তবে তারা এখনও এটির পুরোপুরি উত্তর দেয় না। পদার্থবিজ্ঞান.স্ট্যাকেক্সেঞ্জার / প্রশ্ন / 251385/…
স্টিভ লিন্টন

1
@ মার্কো 36 "ভার্চুয়াল কণাগুলি" ফেনম্যান ডায়াগ্রামের অভ্যন্তরীণ লাইন এবং এর চেয়ে বেশি পুনরায় সংশোধন করা উচিত নয়। কিউএফটি কখনই একটি "ভার্চুয়াল কণা" তে একটি রাষ্ট্রকে বরাদ্দ করে না । তারা সম্পূর্ণ রূপক হয়। আর্নল্ড নিউমায়ার (পদার্থবিজ্ঞান.স্ট্যাকেক্সেঞ্জারএইচএ / 252183 ) এবং তার লিঙ্কগুলির উত্তর দেখুন ।
রবিন একমান

উত্তর:


28

আমি আপনাকে একটি স্বজ্ঞাত উত্তর দিতে যাচ্ছি। মনে রাখবেন, এটি "আসল" উত্তর নয়, কারণ ভার্চুয়াল কণাগুলির সাথে সম্পর্কিত পপ-সায় ব্যাখ্যার চেয়ে হকিং বিকিরণটি খানিকটা জটিল। তবে কিছু স্বজ্ঞাত ন্যায়সঙ্গততা তবুও সম্ভব।

আমি দেখতে পাচ্ছি না যে এই ঘটনাটি কৃষ্ণগহ্বরের বাষ্পীভবনে ভূমিকা রাখে (যেহেতু কণাগুলি ব্ল্যাকহোল থেকে উদ্ভূত হয় না)।

আপনি এখানে একটি মূল পয়েন্ট মিস করছেন।

জুটি তৈরি করা হয়েছিল, সেগুলি ভার্চুয়াল কণা ছিল। এই জুটির একপাশটি ব্ল্যাকহোল দ্বারা শোষিত হওয়ার পরে এবং অন্য দিকে প্রকাশিত হওয়ার পরে, প্রকাশিত অংশটি আসল কণা। সেখানে বিশাল পার্থক্য - ভার্চুয়াল বনাম বাস্তব।

ভার্চুয়াল কণাগুলি যেভাবে আপনি এবং আমার অস্তিত্বের মতই বিদ্যমান নেই। এগুলি খুব অল্প সময়ের জন্যই বিদ্যমান বলে মনে হয়; তারা যত বেশি শক্তিশালী, হেইসেনবার্গ সমীকরণ অনুসারে তাদের ভার্চুয়াল "অস্তিত্ব" -এর অন্তর কম। অনেক উপায়ে তারা কেবল একটি গাণিতিক কৌশল।

শূন্যতার কথা চিন্তা করুন, যেখানে কোনও আসল কণা নেই। আগে, এটি শূন্য। এই মুহুর্তে, একটি ভার্চুয়াল জুটি সংক্ষিপ্তভাবে ফ্লিকার করে, তবে এটি চলে গেছে। ভবিষ্যতে, এটি আবার শূন্য।

এর আগে শক্তি কী ছিল? জিরো। ভবিষ্যতে শক্তি কী? জিরো। ঝাঁকুনির সময় শক্তি কী? ওয়েল, এটি মূলত শূন্য, হাইজেনবার্গের সমীকরণের দ্বারা সীমাবদ্ধতার মধ্যে। নীচের লাইনটি হ'ল ভার্চুয়াল কণা আসে এবং যায় এবং এগুলি কিছু খালি জায়গার শক্তি ভারসাম্যে অবদান রাখে না।

(আমি একটি স্বজ্ঞাত ব্যাখ্যা করার জন্য এখানে ভ্যাকুয়াম শক্তির ধারণাটিকে উপেক্ষা করছি))

তবে আসুন আমরা বলি যে ভার্চুয়াল কণাগুলির একটিটি ব্ল্যাকহোল দ্বারা আটকা পড়ে, তাই এটি এর সমমনা অংশটিকে ধ্বংস করতে পারে না। অন্য কণা বিপরীত দিকে উড়ে এবং ব্ল্যাকহোল থেকে পালিয়ে যায়। সবচেয়ে খারাপ, এটি এখন একটি সত্যিকারের কণা - আমরা হাইজেনবার্গ সমীকরণ দ্বারা অনুমোদিত সময়সীমা অতিক্রম করেছি, সুতরাং যে পালিয়ে যায় সে আর ভার্চুয়াল হয় না।

কীভাবে সেই কণা আসল হয়ে উঠল?

এটি একটি বড় সমস্যা, কারণ ভার্চুয়াল কণাগুলি সংক্ষিপ্তভাবে উপস্থিত হওয়ার জন্য একটি শক্তি বাজেটের প্রয়োজন হয় না, যখন সত্যিকারের কণাগুলি চিরকালের জন্য শক্তি বহন করে। কিছু ভার্চুয়াল জুটিকে নিজেকে ধ্বংস করতে বাধা দিয়েছে এবং উপাদানগুলির মধ্যে একটিটিকে বাস্তব কণার স্থিতিতে উন্নীত করেছে। ভার্চুয়াল জুটির শূন্য শক্তি রয়েছে। আসল কণা যে দূরে যায় তার অ-শূন্য শক্তি থাকে। সেই শক্তি কোথাও থেকে আসতে হবে।

এটি ব্ল্যাকহোল থেকে এসেছে। ভার্চুয়াল থেকে বাস্তবের মধ্যে একটি কণা বৃদ্ধিতে ব্ল্যাকহোল তার কিছু ভর / শক্তি (একই জিনিস) ছেড়ে দেয়। অন্যান্য কণা ধরা পড়েছে - তবে যাইহোক ভার্চুয়াল হওয়ায় এটি আসলে কিছু যায় আসে না।

এই স্বজ্ঞাত ব্যাখ্যাটি যা বলে না তা হ'ল উত্সাহটি কীভাবে ঘটে। আমি জানি, যাদু। কোনওভাবে ভার্চুয়াল কণার একটিতে ব্ল্যাকহোল থেকে একগুণে শক্তি পাওয়া যায় এবং আসল হয়ে যায়।

আবার এটি আসল প্রক্রিয়া নয়। আসল প্রক্রিয়াটি আরও জটিল । এটি কেবল একটি পপ-বিজ্ঞানের রূপকথার গল্প।


সম্পাদনা: বাড়ির কাছাকাছি আঘাত করতে, হকিং রেডিয়েশন আরও বেশিভাবে আনরুহ প্রভাবের নিকটাত্মীয়ের মতো । বলুন কোনও আন্তঃক্ষেত্র পর্যবেক্ষক এই খণ্ডের খণ্ডে খালি জায়গা দেখছেন sees একটি ত্বরণকারী পর্যবেক্ষক একই ভলিউমে খালি জায়গা দেখতে পাবেন না, বরং তার পরিবর্তে ব্ল্যাকবডি বিকিরণ দেখতে পাবেন। এটি উরুহু প্রভাব।

ভাল, মাধ্যাকর্ষণ এবং ত্বরণ সাধারণ আপেক্ষিকতা অনুযায়ী একই জিনিস। সুতরাং একটি ব্ল্যাকহোলের নিকটবর্তী দৃ gra় মাধ্যাকর্ষণ শক্তিশালী ত্বরণের সমান। আনরুহ প্রভাবের অনুরূপ কিছু অবশ্যই সেখানে ঘটতে হবে। এটাই হল হকিং বিকিরণ।

http://backreaction.blogspot.com/2015/12/hawking-radiation-is-not-produced-at.html

সম্পাদনা 2: এই পৃষ্ঠায় অন্যান্য উত্তরগুলি দরকারী বিকল্প পয়েন্ট সরবরাহ করে, তাই তাদেরও পরীক্ষা করে দেখুন।


4
এই "পপ-বিজ্ঞানের রূপকথার গল্প", আপনি নিজেকে বলেছিলেন এটি বেশ মনোরমভাবে পড়া, আমি এমনকি হেসেছি। ধন্যবাদ। তবে এটি হ'ল "আমি যাদু করি" আমি: ভার্চুয়াল কণা কীভাবে এটি আসল অবস্থা লাভ করে (যাদু ছাড়াও) এবং কীভাবে এটি বিএইচ বাষ্পীভবনকে অবদান রাখে, কিছুই নেই যে ব্ল্যাকহোল থেকে বাঁচতে পারে ..
মার্কো ৩৩


আপনি কি সমান অনুপাতে কণা এবং অ্যান্টি-কণা পান?
স্টিভেন গুবকিন

1
আমি অন্তর্দৃষ্টির একটি অংশের সাথে লড়াই করছি - যে কণাটি "সত্য" হিসাবে রূপান্তরিত হয় যখন এমনটি ধরা পড়ে (কেননা সে পালিয়ে যাওয়াটির চেয়ে বেশি কিছু ধ্বংস করে না) "ভার্চুয়াল" থেকে যায় কেন? আমি মনে করি উত্তরটি ভার্সু ভার্চুয়াল বলতে কী বোঝায় তার উত্তর দেওয়া উচিত। আমার অন্তর্নিহিততা আমাকে বলেছে যে পালানো কণা এবং ধরা পড়া কণা একে অপরের মতোই বাস্তব, এবং এইভাবে ব্ল্যাকহোলটি ভর / শক্তি হারিয়ে ফেলতে পারে যার ফলে একজন পালাতে পারে তবে এটি ধরা পড়ে থাকাটির ভর / শক্তি অর্জন করে, ফলে শূন্য সামগ্রিক ক্ষতি।
জেবেন্টলি

@ জেবেন্টলি এটি যাইহোক কেবল একটি গল্প।
ফ্লোরিন আন্দ্রেই

9

এই বক্তৃতা নোটগুলি বিষয়গুলিকে কিছুটা ডিগ্রী, বিশেষত ৩৩-৩৫ স্লাইডে সম্বোধন করে।

কারণ দিগন্তের কাছাকাছিভাবে দৃ war়রূপে warped স্পেসটাইমে, ভ্যাকুয়াম ওঠানামা থেকে তৈরি ভার্চুয়াল কণাগুলি নেতিবাচক শক্তির ঘনত্বকে পরিণত করে।

শক্তি ঘনত্ব = প্রতি ইউনিট ভলিউম energy

এই কণাগুলিতে প্রকৃতপক্ষে ইতিবাচক ভর রয়েছে - যে পালিয়েছে তাকে দেখুন! - তবে তাদের ভর স্থানকালীন সময়ে খুব অদ্ভুতভাবে বিতরণ করা হয়। (কোয়ান্টাম-যান্ত্রিকভাবে বলতে গেলে, কণাগুলির ননজারো ভলিউম থাকে; এটি তরঙ্গ-কণা দ্বৈততার একটি দিক))

নেতিবাচক শক্তির ঘনত্বযুক্ত বিষয়টিকে সাধারণত বহিরাগত পদার্থ বলা হয়

এবং, একটু পরে:

ফ্ল্যাট স্পেসটাইমে কোয়ান্টাম মেকানিকাল ভ্যাকুয়াম ওঠানামার - কোনও শক্তিশালী মহাকর্ষীয় ক্ষেত্র থেকে দূরে - সর্বদা শূন্য নেট শক্তি ঘনত্ব থাকে; তারা কখনও বিদেশী হতে পারে না।

যাইহোক, warped স্পেসটাইমে, ভ্যাকুয়াম ওঠানামা সাধারণ বহিরাগত হয়: তাদের নেট শক্তি ঘনত্ব নেতিবাচক হয়, দূরবর্তী পর্যবেক্ষকের বক্তব্য অনুযায়ী ওঠানামাটির পোশাকের দ্বারা আলোকের প্রতিবিম্ব পর্যবেক্ষণ করে শক্তি ঘনত্ব পরিমাপ করে। বক্রতা যত শক্তিশালী হয় তত শক্তির ঘনত্ব নেতিবাচক হয়।

এটি আমি এখনও অবধি দেখা সেরা ব্যাখ্যা।


এটা অসাধারণ !
ফ্লোরিন আন্দ্রেই

আমি এই বক্তৃতার ব্যাখ্যাও পছন্দ করি। যে কারণে কয়েক ঘন্টা আগে আমি আমার উত্তরে এটি সংযুক্ত করেছি ...

@jakub_d। ওহো দুঃখিত. আপনি কি আমার মুছতে চান?
স্টিভ লিন্টন

1
এটি একটি জটিল ঘটনা। আমি মনে করি উত্তরগুলির বৈচিত্র্য পেয়ে প্রশ্নটি উপকৃত হবে।
ফ্লোরিন আন্দ্রেই

3
আমি কিছু মনে করি না, তবে লোকেরা যদি বক্তৃতার বিভিন্ন বিটগুলি অনুলিপি করে রাখে, শীঘ্রই আমরা এখানে পুরো জিনিসটি করব। :) আমি আগ্রহী ব্যক্তিদের বক্তৃতা নোট পড়তে উত্সাহিত করব, তারা সত্যিই আকর্ষণীয়।

7

হাইজেনবার্গের নীতিটি যতক্ষণ না আপনি সময় মতো সমস্ত কিছু শোধ করেন ততক্ষণ আপনি শক্তি সংরক্ষণ আইনগুলি (যেমন, কোনও কিছুই ছাড়াই কণার জোড় তৈরি করা) অমান্য করতে পারবেন। কণা-অ্যান্টি-পার্টিকেল জুটি যত বড় হবে, তত তাড়াতাড়ি শোধ করতে হবে। ভার্চুয়াল জুটিকে সত্যিকারের জোড়ায় রূপান্তর করা অবৈতনিক energyণের প্রতিনিধিত্ব করার জন্য কিছুটা নেতিবাচক-শক্তি "বহিরাগত বিষয়" (যাই হোক না কেন) উত্পন্ন হিসাবে দেখা যায়। এর শক্তি বিপরীত চিহ্ন সহ জোড়ের সমান। এরপরে এটি একটি কণার পাশাপাশি ব্ল্যাকহোলের মধ্যে পড়ে এবং ব্ল্যাক হোলের ভরকে হ্রাস করে।

ব্ল্যাকহোলের দিগন্তটি কিছু ভার্চুয়াল জোড়া পুনরায় সংশ্লেষ করার পথে পায়, সুতরাং এই রূপান্তরগুলি ভার্চুয়াল-> বাস্তব ঘটবে।

আমি এই বক্তৃতাটি একই ধারণাটি পেয়েছি (আরও বিশদ এবং কম butchered): http://teacher.pas.rochester.edu/Ast102/ বক্তৃতা নোটস / লেকচার 19 / লেকচার 19.pdf


পপ-সায় এখানে তাত্ত্বিকতা দিচ্ছেন, তবে এই সমীকরণের ভারসাম্য বজায় রাখার জন্য কি "বিদেশী বিষয়" আসলেই প্রয়োজনীয়? যদি কোনও ভার্চুয়াল পদার্থ-অ্যান্টিমেটার জোড়া স্ব-ইনস্ট্যান্ট করে এবং নিয়মিত পদার্থ-কণা পালানোর সময় অ্যান্টি-ম্যাটার কণা কৃষ্ণগহ্বরে পড়ে যায়, তবে এটি "কণার" ধ্বংস করা উচিত নয় (হ্যাঁ, আমি জানি যে ব্ল্যাকহোলগুলি অদ্ভুত ™ এবং ব্ল্যাকহোলের কণাগুলি সত্যই কোনও জিনিস নয়, এ কারণেই এটি ব্লকহোল থেকে মূল্যবান), অথবা ব্ল্যাকহোলটি মূলত ভুলে গেছে যে এটি উপাদানগুলির সমন্বয়ে গঠিত উপাদান ব্যবহার করে?
সিডনি

6
@ সিডনি অ্যান্টিমেটারে এখনও ইতিবাচক ভর রয়েছে, যখন পজিট্রন একটি ইলেকট্রনের সাথে দেখা করে, আমরা একে নির্মূল বলি, তবে ফলাফলটি শূন্য নয়, এটি সমতুল্য আপেক্ষিক গণের দুটি গামা ফোটন।

এই উত্তরটি এমন একটি চিন্তাকে উত্সাহিত করেছিল যা আমি ভাগ করে নিতে পারি না। অ্যালকুবিয়েরে ওয়ার্প ড্রাইভের ধারণার জন্য 'বিদেশী বিষয়' বলা দরকার, সুতরাং কোনও ওয়ার্প ড্রাইভ জাম্প-স্টার্ট করার জন্য তাত্ত্বিকভাবে এটির সুবিধা নেওয়া কি সম্ভব? বা এটি কি একই ধরণের বহিরাগত পদার্থ / শক্তি নয়?
কেলি এস ফরাসি 21

1
@ কেলি.ফ্রিঞ্চ আপনি যদি কিছু নেতিবাচক শক্তি ঘনত্বের জিনিস সংগ্রহ করতে চান তবে আপনি উড়ন্ত আয়না এবং পরিবাহী প্লেট (ক্যাসিমিরের প্রভাব দেখুন) দিয়ে খেলানো থেকে ভাল। এটি কোনওভাবেই কাজ করবে না তবে কমপক্ষে এটির জন্য আপনার ব্ল্যাকহোলের দরকার নেই। :)

কেন এটি এমন ঘটবে যে নেতিবাচক এটি ভিতরে যা পড়ে? এই চিত্র দ্বারা একজন গড় নাল প্রভাব আশা করবে।
আলচিমিস্ট

1

বিশেষজ্ঞরা এই বিবরণটির সাথে একমত হবেন কিনা তা আমি জানি না, তবে আমি এটি কীভাবে বুঝতে পারি তা এখানে:

উভয় স্থান এবং ইভেন্ট দিগন্ত স্থির কোয়ান্টাম ওঠানামা মধ্যে। মূলত, ইভেন্ট দিগন্তের ক্ষুদ্রতর ছড়িয়ে রয়েছে। ইভেন্টের দিগন্তগুলি ছড়িয়ে পড়ে এমন পয়েন্টগুলিতে (ব্ল্যাকহোলের গড় ব্যাসার্ধের উপরে), এটিতে স্থানীয় শক্তির উপরে গড় পরিমাণ থাকে। তীব্র মাধ্যাকর্ষণ দ্রুত সেই স্থানীয় ঘাঁটিটিকে আবার নীচে টেনে নিয়ে যায়, পড়ন্ত umpালু ঘটনাটির দিগন্তের বাকি অংশ জুড়ে সেই স্থানীয় শক্তির ঘনত্বকে প্রেরণ করে।

এখন আসুন গর্তটির নিকটে সম্ভাব্য ভার্চুয়াল কণা-জোড়া বিবেচনা করি। যদি কোনও স্থিতিশীল ভার্চুয়াল কণা-জুটি ইভেন্ট দিগন্তের ঠিক উপরে উপস্থিত হয়, তবে তা পুনরুদ্ধার করবে এবং অদৃশ্য হয়ে যাবে বা পুরো জিনিসটি গর্তের মধ্যে টানবে এবং শূন্যে অদৃশ্য হয়ে যাবে। আমাদের একটি ভার্চুয়াল কণা-জোড়া দরকার যা ব্ল্যাকহোল থেকে দূরে প্রায় আলোর গতিতে একটি স্পষ্ট গতিযুক্ত। যদি সেই ভার্চুয়াল কণা-জুটি সম্পূর্ণরূপে পালানোর পক্ষে দ্রুত চলে যায় তবে তারা পুনরায় সংযোগ করে এবং বিলুপ্ত হয়। জিরো নেট ইফেক্ট। আমাদের একটি ভার্চুয়াল কণা-জুড়ি প্রয়োজন যা ব্ল্যাকহোল থেকে প্রায় আলোর গতিতে সরে চলেছে, এবং আমাদের দিগন্তের একটি লম্বালম্বি প্রয়োজন যা কেবল একটি ভার্চুয়াল কণা ধরে। আমি বিশ্বাস করি যে দুটি ভার্চুয়াল কণা থেকে দূরে টানতে, উভয়কে এড়াতে এড়াতে, রিপলটি অবশ্যই চূড়ান্ত নীচের দিকে ত্বকের মধ্যে থাকা উচিত। এবং এখানে মূল অংশটি: কণা জুটির মধ্যে শক্তি-debtণ তীব্রভাবে একে অপরের দিকে টান দেয়। আটকে থাকা কণাটি উপরের দিকে টানছে, কার্যকরভাবে এটি দিগন্তের উপরের দিকে টানছে যা এটি আটকে গেছে। এটি দিগন্ত-লহরগুলির পতনকে ধীরে ধীরে ধীরে ধীরে ধীরে ধীরে পতনশীল রিপলটি ব্ল্যাকহোলের বাকী অংশে ফিরে আসে এমন শক্তি হ্রাস করে।

দুটি ভার্চুয়াল কণা পৃথক করে টানতে প্রয়োজনীয় শক্তি দুটি অ-ভার্চুয়াল কণার সম্মিলিত শক্তির সমান। সুতরাং পতনশীল রিপল দুটি কণার সমান শক্তি হারিয়ে ফেলে এবং গর্তটি একটি কণা খায়। এক পালানো কণার সাথে সমস্ত কিছু সামঞ্জস্য হয়।

ভার্চুয়াল কণা ফোটন বা বিষয়-অ্যান্টিমেটার জোড়া কিনা তা বিবেচনা না করেই এটি একইরূপে কাজ করে বলে আমি বিশ্বাস করি believe


0

এখানে কোয়ান্টাম মেকানিক্সের সাথে সাদৃশ্য। কিউএম-এর একটি কণা একটি অসম্ভব বাধা পেরিয়ে সুড়ঙ্গ করতে পারে, যেভাবে সীসা থেকে ভারী উপাদানগুলি স্ট্রং ফোর্সের বন্ধন থেকে বেঁচে নিউক্লিয়াসের বাইরে থেকে কিছু নিউট্রন "টানেল" রাখতে পারে।

এখানে চিত্র বর্ণনা লিখুন

একটি ছোট ব্ল্যাক হোল একটি কোয়ান্টাম বাধার মতো যা কোনও কণা পালিয়ে যাওয়ার জন্য সুড়ঙ্গ হতে পারে। যত ছোট বাধা (ইভেন্ট হরিজন), তত বেশি টানেল তৈরি করতে সক্ষম হওয়ার সম্ভাবনা তত বেশি। সুতরাং 228 টন ভর দিয়ে একটি মাইক্রো ব্ল্যাকহোল এবং ইভেন্ট হরাইজন 3.4 x 10 ^ -7 ফেমেটোমিটার (আক্ষরিক অর্থে একটি প্রোটনের আকারের 1 মিলিয়ন কম) খুব বেশি দীর্ঘ এবং একেবারে এটির কণাগুলি ধরে রাখবে না। আসলে এটি ঠিক 1 সেকেন্ডের পরে হকিং রেডিয়েশনের একটি ফেটে বিস্ফোরিত হবে ।

একটি সম্পূর্ণ সেন্টিমিটারের ব্যাসার্ধের সাথে একটি বৃহত্তর আর্থ ভর ব্ল্যাকহোল , দীর্ঘতর স্থায়ী হবে: 8 x 10 ^ 50 বছর কারণ কোনও কণার পুরো সেন্টিমিটারের মধ্যে দিয়ে টানেলটি স্বাধীনতা লাভের সম্ভাবনা কম।


উত্স: ত্রিমাত্রিক ব্ল্যাক হোল থেকে কোয়ান্টাম টানেলিং: https://arxiv.org/abs/1306.6380

উত্স: হকিং রেডিয়েশন কোয়ান্টাম এফেক্ট হিসাবে মডেল হয়েছে: http://cscanada.net/index.php/ans/article/view/j.ans.1715787020120502.1817

আমাদের সাইট ব্যবহার করে, আপনি স্বীকার করেছেন যে আপনি আমাদের কুকি নীতি এবং গোপনীয়তা নীতিটি পড়েছেন এবং বুঝতে পেরেছেন ।
Licensed under cc by-sa 3.0 with attribution required.